Combinación en serie de resortes

Cuando un sistema de resorte-masa está conectado verticalmente con dos resortes sin masa en serie cuyas constantes de resorte son k 1 y k 2 a un bloque de masa metro sabemos que fuerzas iguales actúan sobre ambos resortes. Sea esa fuerza durante las oscilaciones F .

Cuando calculamos la constante de resorte efectiva k s , ¿ por qué no decimos que la fuerza neta que actúa sobre el sistema es 2 F ?

Encontrar la fuerza neta que actúa sobre el sistema anterior:

Cuando el bloque está unido, el sistema alcanza la posición de equilibrio a través de desplazamientos X 1 y X 2 .

En equilibrio: 2 F = metro gramo (Dónde F es la magnitud de la fuerza del resorte inicialmente por cada resorte)

Entonces, k 1 X 1 + k 2 X 2 = metro gramo (ecuación 1)

Cuando el sistema se tira hacia abajo, hace oscilaciones, ahora:

Elongación total ser X

Alargamiento en resorte 1 be X 1 y elongación en el resorte 2 ser X 2 .

Fuerza total del resorte = k 1 X 1 k 2 X 2 k 1 X 1 k 2 X 2

Fuerzas totales que actúan sobre el sistema. = k 1 X 1 k 2 X 2 k 1 X 1 k 2 X 2 + metro gramo = metro gramo k 1 X 1 k 2 X 2 + metro gramo
(de la ecuación 1)

Entonces, fuerza total = k 1 X 1 k 2 X 2 = F 1 + F 2 = 2 F (como sabemos que ambas fuerzas son iguales)

Entonces la fuerza neta que actúa sobre el sistema es 2 F

La forma en que calculé la constante de resorte efectiva es:

X = X 1 + X 2

2 F / k s = F / k 1 + F / k 2

2 / k s = 1 / k 1 + 1 / k 2

Pero esa no es una ecuación correcta. ¿Qué hay de malo en tomar la fuerza neta que actúa sobre el sistema como 2 F .

Respuestas (3)

Cuando los resortes (que se supone que no tienen masa) se cuelgan boca abajo, tendrán una extensión cero. así que de hecho

metro gramo = k 1 X + k 2 X
cuando se agrega la masa al sistema.

Escribiste que la fuerza resultante

F r = F 1 + F 2
y llegó a la conclusión de que esto debería ser 2 F como si la fuerza en ambos resortes fuera igual.

Entonces, lo que deberías haber hecho es escribir el desplazamiento resultante

X = X 1 + X 2
de modo que
metro gramo k s = metro gramo k 1 + metro gramo k 2
dónde k s es una constante elástica efectiva. Esto entonces dará
k s = ( 1 k 1 + 1 k 2 ) 1

Los resortes no tienen masa, por lo que la tensión en cualquier punto de cualquiera de los resortes tiene el mismo valor, F . Para los dos resortes individuales, con extensiones X 1 y X 2 , tenemos#

F = k 1 X 1 = k 2 X 2

y cuando se consideran como un solo resorte, tienen una constante de resorte efectiva k tal que

F = k ( X 1 + X 2 ) F = k ( F k 1 + F k 2 ) 1 k = 1 k 1 + 1 k 2

Tenga en cuenta que si unimos dos resortes idénticos en serie (por lo que k 1 = k 2 y X 1 = X 2 ) entonces la constante elástica del resorte articulado es sólo la mitad de la de sus dos componentes. Esto se debe a que produce la misma fuerza F con el doble de la extensión de los resortes individuales, ya que X = 2 X 1 .

Bien, la primera condición para el sistema de resortes en serie es: la fuerza del resorte en todo el sistema es la misma, es decir, la tensión en los resortes es la misma.

por lo tanto
k 1 X 1 = k 2 X 2

k mi ( X 1 + X 2 ) = k 1 X 1
dónde k mi es la constante de resorte equivalente

entonces, si el sistema es vertical, sea la fuerza de gravedad, mg, F

por eso X 1 = F / k 1 , X 2 = F / k 2 y X 1 + X 2 = F / k mi

X 1 + X 2 = F / k 1 + F / k 2 = F / k mi

1 / k mi = 1 / k 1 + 1 / k 2

Ahora abordemos su pregunta, así que le recomiendo que mire nuevamente el diagrama de cuerpo libre. La tensión en todo el sistema es igual a F

Si tiene problemas, también puede intentar relacionar esto con los circuitos de resistencia eléctrica. En una conexión en serie, la corriente es la misma a través de las resistencias, de manera similar, en una conexión de resorte en serie, la tensión es la misma en todas partes. Y en el caso de una conexión en paralelo, la corriente se divide en las ramas, pero cada rama tiene la misma diferencia de potencial, de manera similar, en un sistema de resortes en paralelo, los dos resortes experimentan diferentes tensiones pero se extienden/ contraen por igual.

Para mayor referencia: tengo algunos problemas con los resortes en serie

y

https://physics.stackexchange.com/questions/311111/why-springs-in-series-experience-equal-force#:~:text=In%20series%20circuits%3B%20the%20current,force%20must%20be %20el%20

¿Puede adjuntar un diagrama de cuerpo libre del caso anterior que represente todas las fuerzas que actúan sobre el sistema?